Massimi e minimi senza derivate

Polinomi, disuguaglianze, numeri complessi, ...
Rispondi
bĕlcōlŏn
Messaggi: 145
Iscritto il: 22 gen 2011, 12:56

Massimi e minimi senza derivate

Messaggio da bĕlcōlŏn »

Come al solito, nel questionario della seconda prova di Matematica per il Licei Scientifici, compaiono alcuni problemi di massimo e minimo.
Quesito 1 (Scientifico Tradizionale): Trovare fra tutti i cilindri inscritti in una sfera di raggio $ 60 $ cm quello di volume massimo.
Quesito 2 (Scientifico PNI/Tradizione): Si trovi il punti della curva $ y=\sqrt{x} $ più vicino a quello di coordinate $ (4;0) $.
Quesito 6 (Scientifico PNI): Di tutti i coni inscritti in una sfera di raggio $ 10 $ cm, qual è quello di superficie laterale massima?

Risolveteli senza usare le derivate :)
"Il bon ton è la grazia del saper vivere, la leggerezza dell' esistere." (Lina Sotis, perfidamente elegante)
Mike
Messaggi: 113
Iscritto il: 02 mag 2010, 19:31

Re: Massimi e minimi senza derivate

Messaggio da Mike »

in effetti, il quesito due l'ho risolto nel modo seguente (non sono maturando) :

cercare il punto di quella curva più vicino equivale a cercare la più piccola circonferenza di centro (4,0), che abbia punti in comune con la nostra curva.
consideriamo pertanto un fascio di circonferenze di centro (4,0) e mettiamolo a sistema con la nostra buona curva.
$ y= \sqrt{x} $
$ x^2 +y^2 -8x + 16 = r^2 $

che, sostituendo la prima nella seconda, ci porta a

$ x^2 - 7x + 16 - r^2 = 0 $

A questo punto imponiamo che la circonferenza sia tangente (dimodoché abbia un solo punto in comune), eguagliando il delta a 0:

$ 49 - 64 + 4r^2 = 0 $
$ 4r^2 = 15 $
$ r^2 = \frac{15}{4} $

ora sostituiamo $ r^2 $ nell'equazione e troviamo la soluzione, che è il punto $ (\frac{7}{2}, \sqrt{\frac{7}{2}}) $
Avatar utente
Drago96
Messaggi: 1147
Iscritto il: 14 mar 2011, 16:57
Località: Provincia di Torino
Contatta:

Re: Massimi e minimi senza derivate

Messaggio da Drago96 »

Sto cercando di fare il primo con AM-GM ma non viene! :(
Sono abbastanza sicuro di $V=\pi h(60+h/2)(60-h/2)$ e dunque per massimizzare devo avere ${h+120\over 3}=\sqrt[3]{h(60+h/2)(60-h/2)}$
Ma non riesco a risolvere l'ultima equazione... (e nemmeno wolfram alpha, il che significa che devo aver sbagliato qualcosa, ma non capisco cosa...)
Imagination is more important than knowledge. For knowledge is limited, whereas imagination embraces the entire world, stimulating progress, giving birth to evolution (A. Einstein)
xXStephXx
Messaggi: 472
Iscritto il: 22 giu 2011, 21:51

Re: Massimi e minimi senza derivate

Messaggio da xXStephXx »

Io ho chiamato $ r $ il raggio di base del cilindro.
Quindi ho che $ \sqrt{r^{2}+\frac{h^{2}}{4}}=60 $ e $ \frac{r^{2}h}{3}\pi $ deve essere massimo.

Quindi $ \frac{r^{2}+\frac{h^{2}}{4}}{2} \geq \sqrt{\frac{r^{2}h^{2}}{4}} $
Dopo un po' di conti arrivo a:

$ 4320000\pi \geq \frac{r^{2}h^{2}}{3}\pi $
Il problema è che per trovare il volume dovrei dividere tutto per $ h $, quindi non ottengo un numero preciso.

[Edit] Ho provato ora a fare un sistema a due incognite cercando sia il raggio sia l'altezza mettendo a sistema le due considerazioni fatte, ma siccome mi vengono numeri decimali (calcolatrice), forse è sbagliata anche la considerazione fatta con AM-GM.

OT: ma come mai il codice $ \LaTeX $ mi esce così piccolo?
Avatar utente
Drago96
Messaggi: 1147
Iscritto il: 14 mar 2011, 16:57
Località: Provincia di Torino
Contatta:

Re: Massimi e minimi senza derivate

Messaggio da Drago96 »

Non capisco da dove arriva il "diviso 3" nella formula del volume... non è un cilindro?
Inoltre In AM-GM come mai metti un $h^2$ se devi massimizzare $r^2h$ ?? :?

Comunque qualcuno mi dice dove sbaglio? Perchè ormai dopo un bel po' di tentativi mi sono convinto di quello che ho scritto...

P.S: per gli esponenti fatti da un solo carattere non servono le graffe... ;)
Imagination is more important than knowledge. For knowledge is limited, whereas imagination embraces the entire world, stimulating progress, giving birth to evolution (A. Einstein)
xXStephXx
Messaggi: 472
Iscritto il: 22 giu 2011, 21:51

Re: Massimi e minimi senza derivate

Messaggio da xXStephXx »

Hai ragione il fratto $ 3 $ non ci sta. (perchè l'ho calcolata come se fosse un cono?) xD
Ho messo $ h^2 $ perchè non sapevo come toglierlo. Ora riprovo.
Veluca
Messaggi: 185
Iscritto il: 27 dic 2008, 01:08
Località: Chiavari (Genova)

Re: Massimi e minimi senza derivate

Messaggio da Veluca »

Drago96 ha scritto:Comunque qualcuno mi dice dove sbaglio? Perchè ormai dopo un bel po' di tentativi mi sono convinto di quello che ho scritto...
Il problema è che, come hai fatto am-gm tu, non trovi il volume massimo perchè lasci una "h" nell'AM, prova a farla sparire :)
Testo nascosto:
$\displaystyle V=\frac{\pi}{\sqrt[3]3}\sqrt[3]{h\cdot3\left(60-\frac h2\right)\cdot\left(60+\frac h2\right)}=\frac{\pi}{\sqrt[3]3}\frac{h+180-\frac32h+60+\frac h2}3=\frac{\pi}{\sqrt[3]3}80=\pi\frac{80\sqrt[3]9}3$
Drago96 ha scritto:P.S: per gli esponenti fatti da un solo carattere non servono le graffe... ;)
Ma è buona abitudine metterle per non dimenticarle quando servono :D
Avatar utente
exodd
Messaggi: 728
Iscritto il: 09 mar 2007, 19:46
Località: sulle pendici della provincia più alta d'europa

Re: Massimi e minimi senza derivate

Messaggio da exodd »

@ Veluca: Anche così, non rispetti le condizioni di uguaglianza! :wink:

$ V=\pi\frac{2}{(1+\sqrt{3})(2+\sqrt{3})}[(\frac{1+\sqrt{3}}{2}h)(2+\sqrt{3})(60-\frac{h}{2})(60+\frac{h}{2})]=\pi\frac{2}{(1+\sqrt{3})(2+\sqrt{3})}(\frac{60(3+\sqrt{3})}{3})^3=96000\sqrt{3}\pi $
Tutto è possibile: L'impossibile richiede solo più tempo
julio14 ha scritto: jordan è in realtà l'origine e il fine di tutti i mali in $ \mathbb{N} $
EvaristeG ha scritto:Quindi la logica non ci capisce un'allegra e convergente mazza.
ispiratore del BTA

in geometry, angles are angels

"la traslazione non è altro che un'omotetia di centro infinito e k... molto strano"
Avatar utente
kalu
Messaggi: 297
Iscritto il: 23 nov 2010, 16:52
Località: Pisa

Re: Massimi e minimi senza derivate

Messaggio da kalu »

quesito 1
$ \displaystyle \sqrt[3]{ (\frac{r}{\sqrt{2}})( \frac{r}{\sqrt{2}})( \frac{h}{2})} \le \sqrt{\frac{({\frac{r}{\sqrt{2}}})^2+({\frac{r}{\sqrt{2}} })^2+(\frac{h}{2})^2 }{3}} $; ma $ \displaystyle r^2+ \frac{h^2}{4}=R^2 $, quindi $ \displaystyle V \le \frac{4 \pi R^3 }{\sqrt{27}} $. Si ha l'uguaglianza quando $ h=sqrt{2}r $.

quesito 2
soluzione identica a quella di Mike (ma sospetto che belcolon ne abbia una migliore :wink: )

quesito 6
Applicando il secondo teorema di Euclide (come?) si ottiene che $ r=\sqrt{h(2R-h)} $, quindi $ S_l=\pi h \sqrt{2R(2R-h)} $.
$ \displaystyle \sqrt[4]{ (\frac{h}{2}) (\frac{h}{2}) (\frac{2R}{3}) (2R-h) } \le \frac{(\frac{h}{2}) +(\frac{h}{2})+(\frac{2R}{3})+(2R-h) }{4} $, da cui $ \displaystyle S_l \le \frac{8\pi R^2}{\sqrt{27}} $. Si ha l'uguaglianza quando $ \displaystyle h=\frac{4R}{3} $.
Ultima modifica di kalu il 26 giu 2011, 21:09, modificato 7 volte in totale.
Pota gnari!
xXStephXx
Messaggi: 472
Iscritto il: 22 giu 2011, 21:51

Re: Massimi e minimi senza derivate

Messaggio da xXStephXx »

Non sono stato io a fare il quesito 2 :D
Avatar utente
kalu
Messaggi: 297
Iscritto il: 23 nov 2010, 16:52
Località: Pisa

Re: Massimi e minimi senza derivate

Messaggio da kalu »

Mmm... al quesito 6 ho commesso degli errori, proverò a rimediare. OK, finalmente così dovrebbe andare (avevo scambiato un raggio con un diametro :oops: ). Scusa Mike per averti tolto il merito di aver risolto il 2 :mrgreen: Edito.
Pota gnari!
paga92aren
Messaggi: 358
Iscritto il: 31 lug 2010, 10:35

Re: Massimi e minimi senza derivate

Messaggio da paga92aren »

Detto $h$ l'altezza del cono, $r$ il raggio di base, $l$ il lato e $R$ il raggio della sfera, si ha che:
1) $l^2=2Rh$
2) $r^2=l^2-h^2$
e sapendo che la superficie laterale è $S=\pi lr$ ottengo $S=\pi \sqrt{2Rh^2(2R-h)}$ e massimizzo $h^2(2R-h)=4\frac{h}{2}\frac{h}{2}(2R-h)\leq 4\left( \frac{\frac{h}{2}+\frac{h}{2}+(2R-h)}{3}\right)^3=\frac{32}{27}R^3$ con point of incident $h=\frac{4R}{3}$. Sapendo $S\leq \pi \sqrt{\frac{64}{27}R^4}=\frac{8}{9}\sqrt{3}R^2\pi$
bĕlcōlŏn
Messaggi: 145
Iscritto il: 22 gen 2011, 12:56

Re: Massimi e minimi senza derivate

Messaggio da bĕlcōlŏn »

kalu ha scritto:quesito 1
$ \displaystyle \sqrt[3]{\left(\frac{r}{\sqrt{2}}\right)\left(\frac{r}{\sqrt{2}}\right)\left(\frac{h}{2}\right)} \le \sqrt{\frac{\left({\frac{r}{\sqrt{2}}}\right)^2+\left({\frac{r}{\sqrt{2}} }\right)^2+\left(\frac{h}{2}\right)^2 }{3}} $; ma $ \displaystyle r^2+ \frac{h^2}{4}=R^2 $, quindi $ \displaystyle V \le \frac{4 \pi R^3 }{\sqrt{27}} $. Si ha l'uguaglianza quando $ h=\sqrt{2}r $.
Mike ha scritto: Cercare il punto di quella curva più vicino equivale a cercare la più piccola circonferenza di centro (4,0), che abbia punti in comune con la nostra curva.
consideriamo pertanto un fascio di circonferenze di centro (4,0) e mettiamolo a sistema con la nostra buona curva.
$ y= \sqrt{x} $
$ x^2 +y^2 -8x + 16 = r^2 $
che, sostituendo la prima nella seconda, ci porta a
$ x^2 - 7x + 16 - r^2 = 0 $
A questo punto imponiamo che la circonferenza sia tangente (dimodoché abbia un solo punto in comune), eguagliando il delta a 0:
$ 49 - 64 + 4r^2 = 0 $
$ 4r^2 = 15 $
$ r^2 = \frac{15}{4} $
ora sostituiamo $ r^2 $ nell'equazione e troviamo la soluzione, che è il punto $ (\frac{7}{2}, \sqrt{\frac{7}{2}}) $
paga92aren ha scritto:Detto $h$ l'altezza del cono, $r$ il raggio di base, $l$ il lato e $R$ il raggio della sfera, si ha che:
1) $l^2=2Rh$
2) $r^2=l^2-h^2$
e sapendo che la superficie laterale è $S=\pi lr$ ottengo $S=\pi \sqrt{2Rh^2(2R-h)}$ e massimizzo $h^2(2R-h)=4\frac{h}{2}\frac{h}{2}(2R-h)\leq 4\left( \frac{\frac{h}{2}+\frac{h}{2}+(2R-h)}{3}\right)^3=\frac{32}{27}R^3$ con point of incident $h=\frac{4R}{3}$. Sapendo $S\leq \pi \sqrt{\frac{64}{27}R^4}=\frac{8}{9}\sqrt{3}R^2\pi$
Per avere sott'occhio tutte e tre le soluzioni. In effetti per il secondo esercizio, prendo un generico punto $A(x,\sqrt{x})$ sulla curva con $x\geq 0$. La distanza da $B(4,0)$ è $AB=\sqrt{(x-4)^2+x}=\sqrt{x^2-7x+16}$. Ora basta trovare il "punto più basso della parabola" con $x\geq 0$. L'ascissa del vertice è $\dfrac{7}{2} \geq 0$ e quindi è questo il minimo cercato.

Per il metodo usato nel terzo c'è una simpatica generalizzazione. Dati $n$ numeri reali positivi la cui somma è fissata, il prodotto $\displaystyle\prod_{i=1}^n x_i^{a_i}$, dove gli $a_i$ sono razionali positivi è massimo quando $\dfrac{x_i}{a_i}$ è costante per ogni $i$ (in altre parole sarebbe AM-GM pesata). In quel caso, infatti, il massimo si ha quando $\dfrac{h}{2} = 2R-h \Rightarrow h=\dfrac{4}{3} R$.
"Il bon ton è la grazia del saper vivere, la leggerezza dell' esistere." (Lina Sotis, perfidamente elegante)
Rispondi